¿Por qué tiene sentido hablar de la primera formulación BRST cuantificada de una partícula puntual relativista?

Mi pregunta es sobre la cuantización BRST de una partícula puntual en Polchinski, Vol.1, Sección 4.2.

La cuantificación BRST comienza a partir de la acción efectiva para la integral de trayectoria de calibre fijo. Pero para el enfoque de línea universal de cuantificación de partículas puntuales relativistas, hay un problema.

No hay operadores de posición relativistas y, por lo tanto, no hay estados propios de posición | X para partículas puntuales relativistas. Esta es una de las razones por las que la gente abandonó la "mecánica cuántica relativista" y se inclinó hacia la teoría del campo de cuantificación.

Considerando este problema, la integral de trayectoria de una partícula puntual relativista no tiene sentido porque t , X | t , X no está definido. ¿Tiene algún sentido físico hablar de la cuantificación BRST de una partícula puntual relativista? ¿Es la sección de la cuantización BRST de una partícula puntual relativista en Polchinski simplemente un ejemplo heurístico con fines pedagógicos?

¿En qué parte de la sección correspondiente de Polchinski cree que se necesita la existencia de estados propios de posición? Incluya las partes relevantes en la pregunta para que pueda responderla sin abrir Polchinski. Tenga en cuenta también que el X m ( τ ) Las coordenadas de la partícula puntual relativista no son operadores de posición en el sentido prohibido por la relatividad.
¿Puedes elaborar la última oración de tu comentario?
La teoría "de la partícula puntual relativista" no sabe que es una partícula puntual relativista. Puramente formalmente, es una teoría de 4 campos escalares X m viviendo en un espacio-tiempo unidimensional parametrizado por τ . Hay ciertas suposiciones que entran en los teoremas de no-go para los operadores de posición relativistas que estos campos escalares simplemente no cumplen (y, en consecuencia, no se prestan a ser interpretados como operadores de posición reales). Como dije, edite su pregunta para identificar el paso exacto en el procedimiento BRST que cree que falla.
Ya lo dije en mi post. Pensé que no hay manera de definir | X m > y por lo tanto pensé que no tiene sentido hablar de la cuantización BRST de una partícula puntual relativista.
¿No es más rápido cuantizar la partícula puntual @XiaoyiJing?

Respuestas (2)

Creo que puede haber un malentendido que se aclara en gran medida con el método BRST tal como se presenta en Polchinsky. Evita la discusión en el libro de Ticciati ya que contiene un operador de posición en 4 en lugar de 3 dimensiones. El espacio cinemático de Hilbert obtenido cuantificando la acción

S = d λ ( 1 2 ( X ˙ 2 + metro 2 ) b ˙ C )

produce los operadores X m , PAG m , B y C satisfaciendo las relaciones canónicas de conmutación [ X m , PAG v ] = d v m y { B , C } = 1 . Del teorema de Stone-von Neumann, sabemos que existe una única representación irreducible de estos en el espacio de Hilbert L 2 ( R 4 ) dónde ( X m ψ ) ( X ) = X m ψ ( X ) y ( PAG m ψ ) ( X ) = i m ψ ( X ) . En particular, el delta de Dirac | X = d X , satisfactorio ( X m d X ) ( y ) = y m d ( X y ) = X m d ( X y ) = X m d X ( y ) , es decir X m | X = X m | X , está en la teoría. El conjunto de tales estados son ortogonales.

Ahora, el procedimiento de cuantización canónica también produce el hamiltoniano H = 1 2 ( pag 2 + metro 2 ) y el operador BRST q = C H . Los estados físicos son aquellos para los cuales q ψ = 0 , es decir, aquellos para los que H ψ = 0 . A través de la ecuación de Schrödinger, un estado ϕ : R L 2 ( METRO ) evolucionando en el parámetro λ que es físico entonces satisfaría i d ϕ ( λ ) d λ = H ϕ ( λ ) = 0 , que es precisamente lo que queremos! Los estados no deben depender del parámetro no físico λ . De hecho, un estado ψ L 2 ( METRO ) ya corresponde a toda la línea de mundo de la partícula. Ahora, vemos que aunque tenemos un operador de posición y vectores | X , estos últimos no son físicos ya que no están cerrados por BRST.

De paso, H ψ = 0 es la ecuación de Klein-Gordon. Por lo general, en las clases de QFT se afirma que esta no es una ecuación satisfactoria para una función de onda, ya que la norma obtenida al integrar en tres espacios no se conserva. Sin embargo, ahora vemos que la norma real debe obtenerse integrando todo el espacio de cuatro dimensiones, lo que no genera ningún problema. Por otro lado, la cantidad conservada dada por el Lagrangiano de Klein-Gordon no tiene que interpretarse como una densidad de probabilidad sino como una densidad de carga. Entonces no hay problema con el hecho de que no es definido positivo.

Permítanme señalar que la cuantificación BRST parece ser importante para este análisis. Si uno trata de cuantificar una acción fija de indicador como

S = metro 1 v 2
directamente, se llega a una ecuación de Schrödinger
i d ϕ d t = 2 Δ + metro 2 ϕ ( t )
que no es local. Realmente no sé qué está pasando aquí, pero supongo que es por eso que la gente usa la cuantificación BRST.

Espero no haberme perdido ningún detalle obvio de algo que invalide mis argumentos jajaja. Permítanme finalizar diciendo que

  1. No conozco una forma de introducir interacciones en estas construcciones. Si lo hay, solo debería ser posible para interacciones muy específicas que no violarían la conservación del número de partículas. Es por esto que QFT es muy importante para describir la parte interesante de la teoría relativista de partículas. Sin embargo, se puede hacer una QFT perturbativa en un punto de vista "primero cuantificado" utilizando la acción anterior y los diagramas de Feynman, evitando así la necesidad de, por ejemplo, la acción de Klein-Gordon.
  2. Este punto de vista "primero cuantificado" es muy importante para la teoría de cuerdas, donde la teoría del campo de cuerdas (el análogo de Klein-Gordon para la partícula relativista sin espín) es difícil de obtener, pero el análogo de la acción anterior (la acción de Polyakov) es fácilmente disponibles.

Realmente no entiendo mucho de la teoría de cuerdas, pero esos dos comentarios se hicieron en mi clase.

Omití la parte del espacio de Hilbert con respecto a C y B por simplicidad. La parte que no queremos que provenga de esto se elimina identificando todos los estados exactos de BRST.
¡Ningún problema! También comencé a convencerme de que la descripción que conduce al hamiltoniano no local es equivalente a la obtenida con el procedimiento BRST. La transformación unitaria del primero al espacio físico de Hilbert del segundo se obtiene precisamente mapeando cada función de onda ψ L 2 ( R 3 ) a su vez evolucionado ϕ : R 4 C . Por supuesto, hay que comprobar que ϕ L 2 ( R 4 ) .

Estas equivocado. No hay base ortogonal. | X , t para fijo t en la mecánica cuántica relativista, pero hay un estado | X , t = | X m . En la teoría cuántica de campos, este estado se expresa mediante ϕ ( X ) | 0 , pero este es un estado de una partícula, por lo que puede expresarse en el espacio de una partícula, abarcado por los estados propios de momentos | pag .

| X m = ( 2 π ) 3 / 2 d 3 pag 2 mi pag mi i pag m X m | pag

El punto principal es que el estado | X no implica la existencia de un operador hermitiano X , por lo que este estado puede existir sin la existencia de X -operador. De hecho, este operador no puede existir ya que el hamiltoniano está acotado desde abajo. Puedes ver más al respecto aquí .

si el estado | X m existe, tiene mucho sentido tener integrales de trayectoria sobre trayectorias X m ( τ ) , si calibramos la parametrización al final.

X 2 m | X 1 m = X ( 0 ) = X 1 X ( 1 ) = X 2 [ d X ( τ ) ] d i F F mi S [ X ]

Lo que descubrirás a partir de esa integral de trayectoria es que X 2 | X 1 0 incluso para intervalos similares al espacio, y por la simetría de Lorentz se puede deducir que lo mismo ocurre con la hipersuperficie de simultaneidad. Esto significa que no podemos tomar | X | X , t como base ortogonal para t fijado.

Por eso no hay función de onda. ψ ( X ) descripción de los estados. Para la función de onda de cantidad de movimiento no habrá problema, y ​​una función de onda ϕ ( pag ) es totalmente aceptable. Por ejemplo, PAG es un operador hermitiano para el espacio de Hilbert de una partícula en la teoría libre (libre significa que no se mezcla con estados de múltiples partículas).

En realidad, el enfoque BRST está lejos de ser solo pedagógico. Puede obtener la descripción de la teoría de campos definiendo un operador de campo Ψ ( X m ) e imponer la ecuación de movimiento:

q Ψ = 0

Esto es útil para obtener una generalización no lineal (es decir, cómo establecer interacciones) para partículas en presencia de supersimetría y/o simetría de calibre. Las transformaciones de calibre del campo serán descritas por la generalización de los campos BRST-exactos d Ψ = q Λ + gramo [ Ψ , Λ ] y la ecuación de movimiento será:

q Ψ + gramo Ψ 2 = 0 .

Puedes ver más al respecto aquí .

He visto algunos libros que dicen que los estados locales no se pueden definir en la mecánica cuántica relativista. Teoría de Representaciones de Grupos y Aplicaciones por AO Barut Hasta donde puedo recordar, es de la teoría de representación inducida del álgebra de Poincaré. Quizás no tengas razón.
No se puede hacer un estado localizado en el sentido de | X , t y | y , t siendo ortogonales entre sí, es decir, localizar una partícula en un único punto del espacio. Pero no hay nada que prohíba la existencia del Estado. | X m que tienen la transformación de Poincare correcta. ¡En realidad este estado es único!
Puedes comprobar que este es el estado ϕ ( X ) | 0 , pero también puedes resolver el pag fundamentar y construir el estado
| X m = d 3 pag 2 mi pag mi i pag m X m | pag
Pero del libro "Teoría de las Representaciones y Aplicaciones de Grupos de AO Barut", dice claramente que el estado localizado no se puede definir. Los estados se definen a partir de la representación del álgebra de Poincaré de ese libro. Además, acabo de encontrar otro libro que habla sobre este tema, Una introducción interpretativa a la teoría cuántica de campos en la página 50-51. Está relacionado con el principio de incertidumbre.
Creo que no estás entendiendo lo que dicen. Los estados localizados no son | X m , son estados que son autoestados de X , y estas dos cosas son completamente diferentes.
Hay otro libro que habla de este problema. Teoría cuántica de campos para matemáticos R. TICCIATI en el apartado 1.6. Habla sobre el estado propio de posición en la mecánica cuántica no relativista y por qué no se puede generalizar en la mecánica cuántica relativista.
Solo trata de entender el estado en el que escribí. Realice boost, traducción en este estado y verifique que este estado se comporte bajo estas transformaciones como X m . Nuevamente, este no es un estado localizado.
En el libro anterior "Teoría cuántica de campos para matemáticos R. TICCIATI", el autor dice lo siguiente: El resultado, entonces, es que un operador de posición es inconsistente con la relatividad.
¿Por qué no es un estado localizado? Creo que está localizado en X m , ¿no es así?
El operador X m no existe!!!, el estado | X m NO es un estado propio de X m porque este operador no existe en primer lugar. Lo que define este estado | X m son las propiedades bajo transformaciones de Lorentz. Consulte esto https://physics.stackexchange.com/questions/386645/what-wavefunctions-do-the-creation-operator-of-a-massive-real-scalar-free-field/386651#386651
¿Qué sucede si toma un segmento de posiciones similar al espacio y construye un operador de proyección sumando todos los operadores de proyección de todos los | X m > estados? ¿No es lo mismo que una base local en alguna constante t?
@lurscher este es el punto llamativo. Los Estados | X m en CUALQUIER superficie similar al espacio no son ortogonales entre sí. Esto significa que no proporcionan una base y que los proyectores | X X | y | y y | no conmutará!!! Esto significa que no hay forma de determinar exclusivamente la posición de la partícula. En otras palabras, preguntar si la partícula está en X o preguntando si la partícula está en y son incompatibles mecánicamente cuánticamente.
Y ningún operador hermitiano del tipo X ^ = X X | X X | es posible desde | X no es una base
¿Por qué es un estado de una partícula?
El estado ϕ ( X ) | 0 es un estado de una partícula. Si prefieres empezar con | X sin campos, trabajando la superposición en | pag base, entonces esta será una partícula por construcción;)
Creo que un estado 1p es | pag > .
y todas las superposiciones hechas de él, como
| X m = d 3 pag 2 mi pag mi i pag m X m | pag
¿Cómo veo que este es un estado propio del operador numérico?
Para tener un operador asociado al número de partículas necesitas ir a la Teoría de Campos. Allí, el número de partículas será pag a pag a pag , como puede ver, este estado es un vector propio de él ya que a pag a pag | pag = d pag pag | pag
| pag > es un estado de una partícula definido en la teoría cuántica de campos.